From da9fe2f1b5b914b1923a12b41fb51321758d98f1 Mon Sep 17 00:00:00 2001 From: "W. Trevor King" Date: Mon, 6 May 2013 11:09:43 -0400 Subject: [PATCH] S&Jv8,p27.01: Add missing period to the question --- latex/problems/Serway_and_Jewett_8/problem27.01.tex | 2 +- 1 file changed, 1 insertion(+), 1 deletion(-) diff --git a/latex/problems/Serway_and_Jewett_8/problem27.01.tex b/latex/problems/Serway_and_Jewett_8/problem27.01.tex index 2e738ba..5c1e194 100644 --- a/latex/problems/Serway_and_Jewett_8/problem27.01.tex +++ b/latex/problems/Serway_and_Jewett_8/problem27.01.tex @@ -1,5 +1,5 @@ \begin{problem*}{27.1} -A proton beam in an accelerator carries a current of $125\U{$\mu$A}$ +A proton beam in an accelerator carries a current of $125\U{$\mu$A}$. If the beam is incident on a target, how many protons strike the target in a period of $23.0\U{s}$. \end{problem*} -- 2.26.2